Menu Close

calculate-0-1-e-zx-2-x-2-dx-with-z-from-C-and-Re-z-lt-0-




Question Number 90292 by mathmax by abdo last updated on 22/Apr/20
calculate ∫_0 ^∞  ((1−e^(zx^2 ) )/x^2 )dx with z from C and Re(z)<0
$${calculate}\:\int_{\mathrm{0}} ^{\infty} \:\frac{\mathrm{1}−{e}^{{zx}^{\mathrm{2}} } }{{x}^{\mathrm{2}} }{dx}\:{with}\:{z}\:{from}\:{C}\:{and}\:{Re}\left({z}\right)<\mathrm{0} \\ $$
Commented by mathmax by abdo last updated on 23/Apr/20
let f(z) =∫_0 ^∞  ((1−e^(zx^2 ) )/x^2 )dx ⇒ f^′ (z) =−∫_0 ^∞  e^(zx^2 ) dx  =−∫_0 ^∞  e^(−((√(−z))x)^2 ) dx =_((√(−z))x =u)   −∫_0 ^∞   e^(−u^2 ) (du/( (√(−z))))  =−(1/( (√(−z)))) ∫_0 ^∞  e^(−u^2 ) du =−(π/(2(√(−z)))) ⇒f(z)=π(√(−z)) +c  f(0)=0 =c ⇒f(z) =π(√(−z))   if  z =−α +iβ      (α>0)  −z =α−iβ  =(√(α^2  +β^2 )) e^(−iarctan((β/α)))  ⇒(√(−z))=(α^2  +β^2 )^(1/4)  e^(−(i/2)arctan((β/α)))   ⇒f(z) =^4 (√(α^2  +β^2 )) e^(−(i/2)arctan((β/α)))
$${let}\:{f}\left({z}\right)\:=\int_{\mathrm{0}} ^{\infty} \:\frac{\mathrm{1}−{e}^{{zx}^{\mathrm{2}} } }{{x}^{\mathrm{2}} }{dx}\:\Rightarrow\:{f}^{'} \left({z}\right)\:=−\int_{\mathrm{0}} ^{\infty} \:{e}^{{zx}^{\mathrm{2}} } {dx} \\ $$$$=−\int_{\mathrm{0}} ^{\infty} \:{e}^{−\left(\sqrt{−{z}}{x}\right)^{\mathrm{2}} } {dx}\:=_{\sqrt{−{z}}{x}\:={u}} \:\:−\int_{\mathrm{0}} ^{\infty} \:\:{e}^{−{u}^{\mathrm{2}} } \frac{{du}}{\:\sqrt{−{z}}} \\ $$$$=−\frac{\mathrm{1}}{\:\sqrt{−{z}}}\:\int_{\mathrm{0}} ^{\infty} \:{e}^{−{u}^{\mathrm{2}} } {du}\:=−\frac{\pi}{\mathrm{2}\sqrt{−{z}}}\:\Rightarrow{f}\left({z}\right)=\pi\sqrt{−{z}}\:+{c} \\ $$$${f}\left(\mathrm{0}\right)=\mathrm{0}\:={c}\:\Rightarrow{f}\left({z}\right)\:=\pi\sqrt{−{z}}\:\:\:{if}\:\:{z}\:=−\alpha\:+{i}\beta\:\:\:\:\:\:\left(\alpha>\mathrm{0}\right) \\ $$$$−{z}\:=\alpha−{i}\beta\:\:=\sqrt{\alpha^{\mathrm{2}} \:+\beta^{\mathrm{2}} }\:{e}^{−{iarctan}\left(\frac{\beta}{\alpha}\right)} \:\Rightarrow\sqrt{−{z}}=\left(\alpha^{\mathrm{2}} \:+\beta^{\mathrm{2}} \right)^{\frac{\mathrm{1}}{\mathrm{4}}} \:{e}^{−\frac{{i}}{\mathrm{2}}{arctan}\left(\frac{\beta}{\alpha}\right)} \\ $$$$\Rightarrow{f}\left({z}\right)\:=^{\mathrm{4}} \sqrt{\alpha^{\mathrm{2}} \:+\beta^{\mathrm{2}} }\:{e}^{−\frac{{i}}{\mathrm{2}}{arctan}\left(\frac{\beta}{\alpha}\right)} \\ $$$$ \\ $$

Leave a Reply

Your email address will not be published. Required fields are marked *